Calculate the Fourier series of the function

Click For Summary
The discussion focuses on calculating the Fourier series for the piecewise function defined on the interval [-π, π], which takes the value 1 for [0, π] and -1 for (-π, 0). The function is periodic with a period of 2π and is odd, leading to the conclusion that the constant term a_0 equals zero. Participants suggest using the standard formula for Fourier coefficients, a_n = (1/2π)∫_{-π}^π f(x)e^{-inx}dx, or opting for a half-range sine expansion due to the function's odd symmetry. The conversation emphasizes the need to compute the Fourier coefficients as the next step in the analysis.
Dustinsfl
Messages
2,217
Reaction score
5
Calculate the Fourier series of the function $f$ defined on the interval [\pi, -\pi] by
$$
f(\theta) =
\begin{cases} 1 & \text{if} \ 0\leq\theta\leq\pi\\
-1 & \text{if} \ -\pi < \theta < 0
\end{cases}.
$$
f is periodic with period 2\pi and odd since f is symmetric about the origin.
So f(-\theta) = -f(\theta).
Let f(\theta) = \sum\limits_{n = -\infty}^{\infty}a_ne^{in\theta}.
Then f(-\theta) = \sum\limits_{n = -\infty}^{\infty}a_ne^{-in\theta} = \cdots + a_{-2}e^{2i\theta} + a_{-1}e^{i\theta} + a_0 + a_{1}e^{-i\theta} + a_{2}e^{-2i\theta}+\cdots
-f(\theta) = \sum\limits_{n = -\infty}^{\infty}a_ne^{-in\theta} = \cdots - a_{-2}e^{-2i\theta} - a_{-1}e^{-i\theta} - a_0 - a_{1}e^{i\theta} - a_{2}e^{2i\theta}-\cdots

a_0 = -a_0 = 0

I have solved many Fourier coefficients but I can't think today.

What do I need to do next?
 
Physics news on Phys.org
Why don't you just find the Fourier coefficients by using the usual formula?? That is

a_n=\frac{1}{2\pi}\int_{-\pi}^\pi f(x)e^{-inx}dx
 
Or use the half-range sine expansion since it is an odd function.
 
Question: A clock's minute hand has length 4 and its hour hand has length 3. What is the distance between the tips at the moment when it is increasing most rapidly?(Putnam Exam Question) Answer: Making assumption that both the hands moves at constant angular velocities, the answer is ## \sqrt{7} .## But don't you think this assumption is somewhat doubtful and wrong?

Similar threads

  • · Replies 14 ·
Replies
14
Views
2K
  • · Replies 6 ·
Replies
6
Views
2K
  • · Replies 7 ·
Replies
7
Views
2K
  • · Replies 2 ·
Replies
2
Views
2K
  • · Replies 1 ·
Replies
1
Views
2K
Replies
1
Views
2K
  • · Replies 4 ·
Replies
4
Views
1K
  • · Replies 3 ·
Replies
3
Views
2K
  • · Replies 1 ·
Replies
1
Views
1K
  • · Replies 2 ·
Replies
2
Views
2K